układ z mod

Podzielność. Reszty z dzielenia. Kongruencje. Systemy pozycyjne. Równania diofantyczne. Liczby pierwsze i względnie pierwsze. NWW i NWD.
Awatar użytkownika
rochaj
Użytkownik
Użytkownik
Posty: 411
Rejestracja: 3 lip 2012, o 23:51
Płeć: Mężczyzna
Lokalizacja: komp
Podziękował: 128 razy
Pomógł: 2 razy

układ z mod

Post autor: rochaj »

Niech n będzie liczbą naturalną \(\displaystyle{ (n\ge 6)}\).
Ile trójek \(\displaystyle{ (a,b,c)}\) liczb całkowitych spełnia ten układ
\(\displaystyle{ \begin{cases}a+b+c\equiv 0\pmod n\\ 1\le a<b<c\le n \end{cases}\quad?}\)
Hydra147
Użytkownik
Użytkownik
Posty: 268
Rejestracja: 31 mar 2013, o 20:23
Płeć: Mężczyzna
Lokalizacja: Polska
Podziękował: 1 raz
Pomógł: 82 razy

układ z mod

Post autor: Hydra147 »

Skoro \(\displaystyle{ a<b<c \le n}\), to \(\displaystyle{ a+b+c<3n}\) czyli \(\displaystyle{ a+b+c=n}\), lub \(\displaystyle{ a+b+c=2n}\).
Wiadomo, że równanie:
\(\displaystyle{ x_{1} + x_{2} + x_{3} =m-3}\) ma \(\displaystyle{ {m-1\choose 2}}\) rozwiązań w liczbach całkowitych nieujemnych, czyli równanie \(\displaystyle{ x_{1} + x_{2} + x_{3} =m}\) ma \(\displaystyle{ {m-1\choose 2}}\) rozwiązań w liczbach całkowitych dodatnich. Sprawdźmy ile jest sytuacji, w których \(\displaystyle{ x_{1} = x_{2}}\). Mamy wówczas równanie \(\displaystyle{ 2 x_{1}+ x_{3}=m}\). Dla każdego \(\displaystyle{ x_{1} \in \left\{ 1,2,...,\left[ \frac{m-1}{2} \right] \right\}}\) istnieje \(\displaystyle{ x_{2}}\) spełniające powyższe równanie i jest wyznaczone w sposób jednoznaczny. Jest ich zatem \(\displaystyle{ \left[ \frac{m-1}{2} \right]}\). Ale może też zachodzić \(\displaystyle{ x_{2} = x_{3}}\) oraz \(\displaystyle{ x_{1} = x_{3}}\) czyli razem mamy \(\displaystyle{ 3\left[ \frac{m-1}{2} \right]}\) możliwości. Natomiast jeżeli \(\displaystyle{ m}\) jest podzielne przez \(\displaystyle{ 3}\), to przypadek \(\displaystyle{ x_{1} = x_{2} = x_{3}}\) policzyliśmy \(\displaystyle{ 3}\) razy czyli o \(\displaystyle{ 2}\) za dużo. Ponadto, jeśli kolejność jest istotna (a jest, gydż \(\displaystyle{ a<b<c}\)) to wynik musimy podzielić przez liczbę permutacji czyli \(\displaystyle{ 6}\). Niech \(\displaystyle{ f(n)}\) będzie szukaną liczbą trójek. Korzystając z powyższych własności wyprowadzamy:
\(\displaystyle{ f(n)= \frac{5n^2-18n+g(n)}{12}}\)
gdzie \(\displaystyle{ g(n)}\) to funkcja określona następująco:
\(\displaystyle{ g(n)= \begin{cases} 24 &\text{dla } n\equiv 0 \pmod{6} \\ 13 &\text{dla } n\equiv \pm 1 \pmod{6} \\ 16 &\text{dla } n\equiv \pm 2 \pmod{6} \\ 21 &\text{dla } n\equiv 3 \pmod{6} \end{cases}}\)
Co kończy rozwiązanie zadania.

-- 26 lip 2014, o 12:59 --

Wyliczyłem (mechanicznie jak i za pomocą wzoru), że \(\displaystyle{ f(6)=8; f(7)=11; f(8)=16; f(9)=22}\), wiec wzór chyba jest dobry. Dodatkowo zauważmy, że:
\(\displaystyle{ f(n)= \frac{5n^2-18n+g(n)}{12}= \frac{5n^2-18n+12}{12} + \frac{g(n)-12}{12}}\). Skoro \(\displaystyle{ \frac{g(n)-12}{12} \in \left\langle 0;1 \right\rangle}\), a \(\displaystyle{ f(n)}\) jest zawsze całkowite, to możemy po prostu zapisać \(\displaystyle{ f(n)= \left[ \frac{5n^2-18n}{12}\right] +2}\).

-- 26 lip 2014, o 15:08 --

Trzeba wnieść kolejne poprawki, gdyż moje rozwiązanie zakłada możliwość \(\displaystyle{ c>n}\). Policzymy teraz ile jest takich trójek. Oto ich lista:
\(\displaystyle{ (2n-3,2,1),(2n-4,3,1),(2n-5,4,1),(2n-5,3,2),(2n-6,5,1),(2n-6,4,2)...}\)
Jest tylko \(\displaystyle{ 1}\) trójka, w której \(\displaystyle{ c=2n-3,2n-4}\) ale już \(\displaystyle{ 2}\), w których \(\displaystyle{ c=2n-5,2n-6}\).Widzimy zatem, że jeżeli \(\displaystyle{ n}\) jest parzyste, to musimy odjąć \(\displaystyle{ 1+1+2+2+...+ (\frac{n}{2}-2) + (\frac{n}{2}-2)+(\frac{n}{2}-1)= \frac{n^2-4n+4}{4}}\). Analogowo rozumujemy dla \(\displaystyle{ n}\) nieparzystego dostając do odjęcia \(\displaystyle{ 1+1+2+2+...+ \frac{n-3}{2} + \frac{n-3}{2}= \frac{n^2-4n+3}{4}}\). Dostajemy zatem nową funkcję \(\displaystyle{ f}\):
\(\displaystyle{ f(n)= \begin{cases} \frac{n^2-3n+6}{6} &\text{gdy } 3|n \\ \frac{n^2-3n+2}{6} &\text{w przeciwnym wypadku } \end{cases}}\).
Łatwo zauważyć, że funkcja \(\displaystyle{ f}\) daje się również przedstawić w postaci \(\displaystyle{ f(n)=$\left\lceil\dfrac{(n-1)(n-2)}{6}\right\rceil$}\). Istotnie, gdy \(\displaystyle{ n=3k}\), to zachodzi
\(\displaystyle{ $\left\lceil\dfrac{(3k-1)(3k-2)}{6}\right\rceil$=$\left\lceil 3 \frac{k(k-1)}{2}+ \frac{1}{3} \right\rceil$= \dfrac{3k(k-1)}{2}+1= \frac{(3k)^2-3(3k)+6}{6}}\) , zaś gdy \(\displaystyle{ n}\) jest niepodzielne przez \(\displaystyle{ 3}\), to zachodzi po prostu równość \(\displaystyle{ $\left\lceil\dfrac{(n-1)(n-2)}{6}\right\rceil$=\dfrac{(n-1)(n-2)}{6}=\frac{n^2-3n+2}{6}}\).
robertm19
Użytkownik
Użytkownik
Posty: 1847
Rejestracja: 8 lip 2008, o 21:16
Płeć: Mężczyzna
Lokalizacja: Staszów/Warszawa
Podziękował: 7 razy
Pomógł: 378 razy

układ z mod

Post autor: robertm19 »

A ten przypadek \(\displaystyle{ a+b+c=2n}\) uwzględniłeś?
Powstanie tych funkcji też jest nie jasne.
A co z \(\displaystyle{ a,b>n}\)?
Hydra147
Użytkownik
Użytkownik
Posty: 268
Rejestracja: 31 mar 2013, o 20:23
Płeć: Mężczyzna
Lokalizacja: Polska
Podziękował: 1 raz
Pomógł: 82 razy

układ z mod

Post autor: Hydra147 »

Tak przypadek \(\displaystyle{ a+b+c=2n}\) uwzględniłem. Dodaję do tego liczbę trójek z przypadku \(\displaystyle{ a+b+c=n}\), a odejmuję liczbę trójek, w których \(\displaystyle{ c>n}\). A skoro \(\displaystyle{ a<b<c}\), to na pewno \(\displaystyle{ a,b<n}\). Jeśli masz jakieś pytania odnośnie konkretnych przejść, za półtorej godzinki postaram się na nie odpowiedzieć. Wynik jest dobry, bo zgadza się z odpowiedzią Rochaja.
robertm19
Użytkownik
Użytkownik
Posty: 1847
Rejestracja: 8 lip 2008, o 21:16
Płeć: Mężczyzna
Lokalizacja: Staszów/Warszawa
Podziękował: 7 razy
Pomógł: 378 razy

układ z mod

Post autor: robertm19 »

Ok, jestem ciekaw.
Hydra147
Użytkownik
Użytkownik
Posty: 268
Rejestracja: 31 mar 2013, o 20:23
Płeć: Mężczyzna
Lokalizacja: Polska
Podziękował: 1 raz
Pomógł: 82 razy

układ z mod

Post autor: Hydra147 »

Ok, już jestem. Napisz mi, których przejść konkretnie nie rozumiesz, to Ci je wyjaśnię.
robertm19
Użytkownik
Użytkownik
Posty: 1847
Rejestracja: 8 lip 2008, o 21:16
Płeć: Mężczyzna
Lokalizacja: Staszów/Warszawa
Podziękował: 7 razy
Pomógł: 378 razy

układ z mod

Post autor: robertm19 »

No pokaż jak tą funkcję konstruujesz.
Hydra147
Użytkownik
Użytkownik
Posty: 268
Rejestracja: 31 mar 2013, o 20:23
Płeć: Mężczyzna
Lokalizacja: Polska
Podziękował: 1 raz
Pomógł: 82 razy

układ z mod

Post autor: Hydra147 »

Zakładam, że wszystko do momentu użycia przeze mnie poraz pierwszy symbolu \(\displaystyle{ f}\) rozumiesz.
Rozbiłem problem ogólny na \(\displaystyle{ 4}\) przypadki. \(\displaystyle{ 3|n \wedge 2|n}\); \(\displaystyle{ 3 \! \not | \; n \wedge 2 \! \not | \; n}\); \(\displaystyle{ 2|n \wedge 3 \! \not | \; n}\); \(\displaystyle{ 2 \! \not | \; n \wedge 3|n}\). Zauważ, że odpowiadają one (odpowiednio) przystawaniom do \(\displaystyle{ 0, \pm 1, \pm 2, 3}\) modulo \(\displaystyle{ 6}\). Co musimy zrobić aby uzyskać wynik:
1. Obliczyć ile jest trójek liczb całkowitych dodatnich \(\displaystyle{ a,b,c}\) spełniających \(\displaystyle{ a+b+c=n}\) oraz \(\displaystyle{ a+b+c=2n}\) (jest ich odpowiednio \(\displaystyle{ {n-1\choose 2}}\) oraz \(\displaystyle{ {2n-1\choose 2}}\)).
2. Odjąć od tej liczby ilość rozwiązań w których co najmniej \(\displaystyle{ 2}\) zmienne są sobie równe (musimy zatem odjąć \(\displaystyle{ 3\left[ \frac{n-1}{2} \right] + 3\left[ \frac{2n-1}{2} \right])}\). I tu pojawia się różnicowanie na parzyste i nieparzyste. Jeśli bowiem \(\displaystyle{ n}\) nieparzyste, to musimy odjąć \(\displaystyle{ \frac{9n-9}{2}}\), a gdy parzyste to \(\displaystyle{ \frac{9n-12}{2}}\).
3. Jeżeli \(\displaystyle{ 3|n}\), to może zajść także \(\displaystyle{ a=b=c}\). Przypadek ten w podpunkcie \(\displaystyle{ 2.}\) odjęliśmy trzykrotnie, musimy zatem teraz dodać dwójkę (jeśli \(\displaystyle{ 3|n}\)). Robi się więc podział na podzielne i niepodzielne przez \(\displaystyle{ 3}\).
4. To co otrzymaliśmy do tej pory to ilość trójek \(\displaystyle{ (a,b,c)}\) dodatnich liczb całkowitych takich, że \(\displaystyle{ a \neq b \neq c \neq a}\) oraz \(\displaystyle{ a+b+c=n}\) lub\(\displaystyle{ a+b+c=2n}\). My chcemy jednak, by \(\displaystyle{ c>b>a}\). Otrzymany wynik musimy więc podzielić przez liczbę permutacji zbioru \(\displaystyle{ 3}\)-elementowego czyli \(\displaystyle{ 6}\).
5.Tak otrzymany wynik prowadzi nas do tej pierwszej funkcji \(\displaystyle{ f}\) lecz nie jest ona poprawna, gdyż może się zdarzyć, że \(\displaystyle{ c>n}\). To ile jest takich trójek (w zależności od parzystości \(\displaystyle{ n}\)) liczyłem już w poprzednim poście.
Widzimy, że dla \(\displaystyle{ n}\) parzystych i \(\displaystyle{ n}\) nieparzystych otrzymujemy taki sam wynik. Znaczenie ma tylko podzielność przez \(\displaystyle{ 3}\). Prowadzi nas to do tej drugiej definicji funkcji \(\displaystyle{ f}\). Jest ona poprawna ale trochę zbyt skomplikowana. Za pomocą przekształceń wyprowadzamy wzór \(\displaystyle{ f(n)=$\left\lceil\dfrac{(n-1)(n-2)}{6}\right\rceil$}\), który jest ładny, miły i przyjemny .
robertm19
Użytkownik
Użytkownik
Posty: 1847
Rejestracja: 8 lip 2008, o 21:16
Płeć: Mężczyzna
Lokalizacja: Staszów/Warszawa
Podziękował: 7 razy
Pomógł: 378 razy

układ z mod

Post autor: robertm19 »

Chwileczke, skąd założenie o rozbiciu na takie 4 przypadki?
Hydra147
Użytkownik
Użytkownik
Posty: 268
Rejestracja: 31 mar 2013, o 20:23
Płeć: Mężczyzna
Lokalizacja: Polska
Podziękował: 1 raz
Pomógł: 82 razy

układ z mod

Post autor: Hydra147 »

Chodzi o to, że w zależności od parzystości \(\displaystyle{ n}\) liczba, jaką odejmujemy w podpunkcie 2. jest inna, a od podzielności przez \(\displaystyle{ 3}\) zależy czy w podpunkcie 3. dodamy tę dwójkę czy nie. Niepotrzebnie napisałem o tym rozbiciu na \(\displaystyle{ 4}\) przypadki na samym początku. Z resztą później i tak zwija się to do dwóch, gdyż niezależnie od parzystości \(\displaystyle{ n}\) otrzymujemy te same wyniki.
ODPOWIEDZ